0
$\begingroup$

Say $\mathfrak{A}$ is a seperable $C^*$ algebra, the space $K$ of states on $\mathfrak{A}$ is compact and convex. Let $\Gamma$ be a countable discrete group acting on $\mathfrak{A}$ via $*$-hom. This gives us an action on $K$, namely $$\gamma.\tau(a)=\tau(\gamma^{-1}a)$$ for every $\tau\in K$. Now given a probability measure $\mu\in Prob(\Gamma)$, we can define the convex compact set of $\mu$-stationary states $$\{\tau\in K:\sum_{\gamma\in\Gamma}\mu(\gamma)\gamma.\tau=\tau\}$$ I am interested in the extreme points of this set. I was not able to find anything online in this regard. One can also define the operator system $$\{a\in\mathfrak{A}:\sum_{\gamma\in\Gamma}\mu(\gamma)\gamma.a=a\}$$ Are there any known relations between the state space of this operator system and restrictions of the $\mu$-stationary states?

$\endgroup$
13
  • $\begingroup$ @Yemonisloggedoutrightnow this is an operator system (closed under star since the action is by * hom). Fixed $\endgroup$ Commented Oct 27 at 16:55
  • $\begingroup$ At least there isn’t a one-to-one correspondence, if that’s what you’re asking for. For example, let $\Gamma=\mathbb{Z}$ act on $\mathfrak{A}=C([0,1])^{\otimes\mathbb{Z}}$ via Bernoulli shift and let $\mu$ be the Dirac delta at one of the generators on $\Gamma$. Then the operator system you wrote down is just $\mathbb{C}$, so it has exactly one state. But there are plenty of $\mu$-stationary measures (for each state on $C([0,1])$ the infinite tensor state on $\mathfrak{A}$ is even $\Gamma$-invariant, for example). $\endgroup$ Commented Oct 27 at 20:12
  • $\begingroup$ Instead of considering the operator system of $\mu$-stationary elements of $\mathfrak{A}$, you should consider the operator system of $\mu$-stationary elements of the double dual of $\mathfrak{A}$. The space of normal states on that operator system is actually naturally in one-to-one correspondence with $\mu$-stationary elements of $K$ ($\mu$-stationary measures in the terminology you used). $\endgroup$ Commented Oct 27 at 20:22
  • $\begingroup$ @DavidGao Have you got a reference? $\endgroup$ Commented Oct 28 at 6:57
  • $\begingroup$ I don't have a reference. I just thought of an argument in my head. But now that I thought about this again, I realized my argument is wrong. I mistakenly thought that there would be a normal conditional expectation from $\mathfrak{A}^{\ast\ast}$ onto the sub-operator system consisting of $\mu$-stationary elements, but that isn't correct. There is a conditional expectation, but it doesn't have to be normal. So, the correct statement should have been the space of $\mu$-stationary states on $\mathfrak{A}$ is only a retract (via affine maps) of the space of normal states on the operator system... $\endgroup$ Commented Oct 31 at 7:08

0

You must log in to answer this question.

Start asking to get answers

Find the answer to your question by asking.

Ask question

Explore related questions

See similar questions with these tags.